2020 AMC 12A Problems/Problem 17

Revision as of 02:01, 2 February 2020 by Lopkiloinm (talk | contribs) (Solution 1)

Problem 17

The vertices of a quadrilateral lie on the graph of $y=\ln{x}$, and the $x$-coordinates of these vertices are consecutive positive integers. The area of the quadrilateral is $\ln{\frac{91}{90}}$. What is the $x$-coordinate of the leftmost vertex?

$\textbf{(A) } 6 \qquad \textbf{(B) } 7 \qquad \textbf{(C) } 10 \qquad \textbf{(D) } 12 \qquad \textbf{(E) } 13$

Solution 1

Let the $x$-intercept be $n.$

Realize that by the shoelace formula the area of the triangle must be $-\lnn+\ln{n+1}+\ln{n+2}-\ln{n+3}.$ (Error compiling LaTeX. Unknown error_msg) That equals to $\ln\frac{(n+1)(n+2)}{(n(n+3)}.$

$\ln\frac{(n+1)(n+2)}{(n(n+3)} = \ln\frac{n^{2}+3n+2}{n^{2}+3n}$

$\ln\frac{n^{2}+3n+2}{n^{2}+3n} = \frac{91}{90}$

$\ln\frac{n^{2}+3n+2}{n^{2}+3n} = \frac{182}{180}$

$n^{2}+3n = 180$

$n^{2}+3n-180 = 0$

$(n-12)(n+15) = 0$

The $x$-intercept is $\boxed{\textbf{(D) } 12.}$~lopkiloinm.